subject
Physics, 16.07.2020 21:01 PrincessKeliah5538

A pair of long, rigid metal rods each of length L, lie parallel to each other on a perfectly smooth table. Their ends are connected by identical, very light conduction springs of force constant k each and negligible unstretched length. If the current I runs through the circuit, the springs will stretch. At what separation will the rods remain at rest? Assume that k is large enough so that the separation of rods will be much less than L. Express your answer in terms of I, L, k and appropriate constants (μo,π).

ansver
Answers: 1

Other questions on the subject: Physics

image
Physics, 22.06.2019 12:30, cxmochaa
If you place a magnet under a clear dish, and sprinkle iron fillings over it, above what part of the magnet will most of the fillings gather
Answers: 2
image
Physics, 22.06.2019 20:30, IHATEIXL
Why does karst topography exist in some geographic location? a. few climates have enough oxygen b. most regions have rock layers other than limestone c. in most regions, the soil is too thin d. the water is too acidic enough in most places
Answers: 2
image
Physics, 22.06.2019 23:30, kwilson5328
Which system of units is used by only a small number of countries in the world, including the u. s.
Answers: 3
image
Physics, 23.06.2019 05:00, sandram74691
City b is south of city a, and the 200 mile trip by car takes 4 hours. the cities are 120 miles apart in a straight line. what is the velocity of a trip from city a to city b?
Answers: 2
You know the right answer?
A pair of long, rigid metal rods each of length L, lie parallel to each other on a perfectly smooth...

Questions in other subjects:

Konu
English, 26.01.2020 17:31